Está en la página 1de 44

2

Ejercicio 2: Identidad de Lagrange y desigualdad de Cauchy-Schwarz

Sean z1 , . . . , zn números complejos.

(a) Prueba por inducción que


2
X n n n−1 X n
z 2 + 2
X X
z j = j Re(zk z j ) . (0.1)

j=1 j=1 k=1 j=k+1

(b) Deduce, a partir de la igualdad (0.1), la identidad de Lagrange


2    
X n X n   n  n−1
z j w j =  z j 2   w j 2  − 2
X X 2
zk w j − z j wk .
  

 
   
 (0.2)

j=1 j=1 j=1 16k< j6n

(c) Deduce, a partir de la igualdad (0.2), la desigualdad de Cauchy-Schwarz


2    
X n X n   X n 
 2
z j w j 6  z j   w j  .
2 

 
   
j=1 j=1 j=1

Solución.
(a) Para n = 2 tenemos:

|z1 + z2 |2 = (z1 + z2 )(z1 + z2 ) = |z1 |2 + |z2 |2 + z1 z2 + z1 z2 = |z1 |2 + |z2 |2 + 2Re(z1 z2 )

Ası́, se cumple para n = 2. Ahora veamos el paso inductivo:


n+1 2
 n+1   n+1   n   n 
X X  X  X  X  Xn Xn
zk =  zk   zk  =  zk   zk  + zn+1
        zk + |zn+1 | + zn+1
2
zk
k=1 k=1 k=1 k=1 k=1 k=1 k=1
n+1 n−1 Xn
 n
 n+1 n−1 X n n
X X  X  X X X
= |zk |2 + 2 Re(zk z j ) + 2Re zn+1 zk  = |zk |2 + 2 Re(zk z j ) + 2 Re(zk zn+1 )
k=1 k=1 j=k+1 k=1 k=1 k=1 j=k+1 k=1
n+1
X n X
X n+1
= |zk |2 + 2 Re(zk z j )
k=1 k=1 j=k+1
(b) Primero notemos que:

|zk w j − z j wk |2 = |zk w j |2 + |z j wk |2 − 2Re(zk wk z j w j )

Si usamos la fórmula (a) obtenemos:


2
X n Xn n−1 X
X n
zk wk = |zk wk |2 + 2 Re(z j w j zk wk ).
k=1 k=1 k=1 j=k+1

Si remplazamos lo anterior obtenemos:


2
X n Xn n−1 X
X n
zk wk = |zk wk | +
2
|zk w j |2 + |z j wk |2 − |zk w j − z j wk |2
k=1 k=1 k=1 j=k+1
n
X X X
= |zk wk |2 + |zk w j |2 + |z j wk |2 − |zk w j − z j wk |2
k=1 1≤k< j≤n 1≤k< j≤n
3
 n  n 
X  X  X
=  |zk |2   |wk |2  − |zk w j − z j wk |2
k=1 k=1 1≤k< j≤n

− z j wk |2
P
(c) La desigualdad se deduce de (b) notando que:0 ≤ 1≤k< j≤n |zk w j


Ejercicio 3: Continuidad de la estructura compleja

(a) Prueba que las funciones

w(z) = Re(z) , w(z) = Im(z) , w(z) = |z| , w(z) = z ,

son continuas en C.

(b) Suponga que z 7→ f (z) es una función continua en un dominio Ω ⊆ C. Prueba que las funciones

w(z) = Re f (z) , w(z) = Im f (z) , w(z) = f (z) , w(z) = f (z) ,
 

son continuas en Ω.

Solución.
(a) Para ver que dichas funciones son continuas, basta probar que | f (z + h) − f (z)| → 0si|h| → 0.
Notemos que: |Re(z + h) − Re(z)| ≤ |z + h − z| = |h|, |Im(z + h) − Im(z)| ≤ |z + h − z| = |h|, ||z + h| − |z|| ≤
|z + h − z| = |h| y |z + h − z| = |h| = |h|. De lo que se deduce la continuidad.
(b) Las primeras tres funciones son composición de funciones continuas, por lo que son continuas en
Ω. La última función es continua por lo antes mencionado, pero en Ω∗ := {z ∈ C : z ∈ Ω}


Ejercicio 4: Continuidad del argumento principal


Sea !
Im(z)
Arg(z) := 2 arctan , z ∈ C − {0} (0.3)
Re(z) + |z|
la funcin argumento principal.
(a) Prueba que Arg(z) es continua en C − (−∞, 0] y es discontinua en (−∞, 0).

(b) Prueba que la funcióne (0.3) coincide con la función


 y


 arctan si x>0



 x

 y
+π x < 0, y > 0

arctan si



x





y
  
Arg(x + i y) := 

−π x < 0, y < 0

arctan si


 x
π



+ x = 0, y > 0

si



2



π



x = 0, y < 0 .

−

 si
2
4

Solución.
(a) La función es continua (en C \ (−∞, 0]) por ser composición de funciones continuas. Ahora veamos
que es discontinua en (−∞, 0): Sea x > 0, definamos la sucesión zn := −x − i n1 . Claramente Arg(zn ) →
−π, zn → −x, pero Arg(−x) = π, por lo que no es continua en x.  
(b) Para ver la primera igualdad, basta ver que arctan(x) = 2 arctan √x . Ası́:
1+ 1+x2
 
 y   
y y
x
= =
   
2 arctan  2 arctan 
 
 arctan
x + x 2 + y2
q p
x
  
 1 + 1 + y2 
x 2

Para la segunda igualdad, recordemos que si x > 0, arctan(x) = π2 − arctan 1x . Por lo que:
 

 x + x2 + y2 
   p   
y −y/x
 = π − 2 arctan   = π − 2 arctan 
   
2 arctan       
x + x 2 + y2 1 + 1 + (y/x)2
p p
y
y
= π + arctan
x

Para la tercera, basta usar la segunda y para la cuarta y la quinta, basta evaluar la función.

PONTIFICIA UNIVERSIDAD CATÓLICA DE CHILE
FACULTAD DE MATEMÁTICAS
DEPARTAMENTO DE MATEMÁTICAS

Clase de ejercicios 10 - 19/10/2015 ∗

Curso: Variable Compleja I


Sigla: MAT2705 - Segundo Semestre 2015
Profesor: Giuseppe De Nittis (gidenittis@mat.puc.cl)
Ayudante: Victor Cañulef Aguilar (vacanulef@uc.cl)

Ejercicio 1
Pruebe que
Z +∞
cos(ax)
dx = π e−a , a > 0.
−∞ x2 + 1
Solución.
Consideramos la función
e i az
f (z) :=
z2 + 1
y notamos que
cos(ax)
Re f (z) = 2 , si Im(z) = 0 .

x +1
Por otro lado |e i az | = |e i a(x+ i y) | = e−ay 6 1 si y > 0. Ası́ | f (z)| ∼ |z|−2 para |z| suficientemente grande
y Im(z) > 0. Ahora consideremos el camino ΩR := ΓR ∪ [−R, R] con ΓR := {Re i θ | θ ∈ [0, π], R > 1}.
Notemos que
X
f (z) dz = 2π i Resz j ( f ) = 2π i Res i ( f )
ΩR j

ya que el único polo de f dentro ΩR está en z = i . Por otra parte


e i az 1
Res i ( f ) := lim (z − i) f (z) = lim =
z→ i z→ i z+ i 2 i ea
ya que el polo es de orden 1. Por último veemos que
Z +R
π
Z
2π i
f (x) dx + f (z) dz = f (z) dz = a
= a .
−R ΓR ΩR 2i e e
Además
πR
Z Z Z Z
| dz| | dz|
f (z) dz 6 | f (z)| | dz| 6 6 6 2
ΓR |z + 1|
2 2
ΓR |z | − 1 R −1

ΓR ΓR

∗Soluciones y notas estarán disponibles en la página web:


https://gdenittis.wordpress.com/courses/variable-compleja-i-mat2705/clase-de-ejercicios
1
2

ya que |z2 | − 1 = |z2 | − | − 1| 6 |z2 − (−1)| = |z2 + 1|. Entonces,


Z +R
π πR
a
= lim f (z) dz = lim f (x) dx + lim 2
e R→+∞ ΩR R→+∞ −R R→+∞ R − 1
| {z }
=0
es decir
+∞ +R
π
Z Z
cos(ax)
dx := lim f (x) dx = .
−∞ x2 + 1 R→+∞ −R ea


Ejercicio 1
Pruebe que
Z 2π !
cos(θ) 2
dθ = 2π 1 − √ .
0 2 + cos(θ) 3
Solución.
Las integrales de 0 a 2π de términos trigonométricos se pueden llevar a integrales de funciones mero-
morfas en el cı́rculo unitario. Sea z = e i θ y observamos que
dz
dz = i e i θ dθ = i z dz ⇒ dθ =
iz
y
z+z z + 1z z2 + 1
cos(θ) = = = .
2 2 2z
Ası́
z2 + 1
Z 2π
cos(θ) 1 dz
dθ =
0 2 + cos(θ) |z|=1 2z
z 2 +1
2 + 2z i z

1 z2 + 1
= dz
i |z|=1 z(z2 + 4z + 1)
z2 + 1
X !
= 2π Resz j .
|z j |<1
z(z2 + 4z + 1)

La función integranda tiene poles z0 := 0 y z± := −2 ± 3 (z± son las raı́ces de z2 + 4z + 1 = 0). Sin

embargo sólo z0 = 0 y z+ = 3 − 2 se encuentran dentro de la curva |z| = 1. Calculamos los residuos:
z2 + 1 z2 + 1
!
Resz0 := lim z = 1;
z(z2 + 4z + 1) z→0 z(z2 + 4z + 1)

z2 + 1 z2 + 1 z2+ + 1
!
2
Resz+ := lim (z − z+ ) = = −√ .
z(z + 4z + 1)
2 z→z+ z(z − z+ )(z − z− ) z+ (z+ − z− ) 3
Por lo tanto Z 2π !
cos(θ) 2
dθ = 2π 1 − √ .
0 2 + cos(θ) 3

PONTIFICIA UNIVERSIDAD CATÓLICA DE CHILE
FACULTAD DE MATEMÁTICAS
DEPARTAMENTO DE MATEMÁTICAS

Clase de ejercicios 11 - 09/11/2015 ∗

Curso: Variable Compleja I


Sigla: MAT2705 - Segundo Semestre 2015
Profesor: Giuseppe De Nittis (gidenittis@mat.puc.cl)
Ayudante: Victor Cañulef Aguilar (vacanulef@uc.cl)

Ejercicio 1
Sea Bδ (z0 ) := {z ∈ C | |z − z0 | < δ} un disco abierto y f analı́tica en Bδ (z0 ) − {z0 }. Considere el arco
Cε := {z0 + εe i θ | θ ∈ [0, α]} y 0 < ε < 1. Si z0 es un polo de orden 1 de f pruebe que
Z
lim f (z) dz = i α Resz0 ( f ) .
ε→0 Cε

Solución.
Sea a− := Resz0 ( f ). Entonces,
a−
f (z) = + g(z)
z − z0
con g analı́tica en Bδ (z0 ). Por lo tanto
Z Z Z
dz
f (z) dz = a− + g(z) dz .
Cε Cε z − z0 Cε

Por otra parte Z Z


| dz| = ε max |g(z)| α ,

g(z) dz 6 max |g(z)|
Cε |z|<δ Cε |z|<δ
o sea Z
lim g(z) dz = 0 .
ε→0 Cε
Además Z Z α
dz 1
= i εe i θ dθ = i α .
Cε z − z0 0 εe i θ


Ejercicio 2
Pruebe que

π2
Z
log(x)
dx = .
0 x2 − 1 4
∗Soluciones y notas estarán disponibles en la página web:
https://gdenittis.wordpress.com/courses/variable-compleja-i-mat2705/clase-de-ejercicios
1
2

Solución.
Sea
Log(z)
f (z) := (valor principal) .
z2 − 1
Note que f tiene dos polos z = 0 y z = −1 (en z = 1 la singularidad es removible). En z = −1 el polo
es de orden 1 ya que Log(−1) = i π , 0. Ahora consideremos la siguiente curva cerrada CR,r . Sean
R > r > 0 constantes positivas y considere los arcos
- γ1 = γ1 (r, R) es el intervalo real [r, R];
- γ2 = γ2 (R) es el semicı́rculo de radio R, centrado en el origen, que se encuentra en el semiplano
superior y orientado positivamente;
- γ3 = γ3 (r, R) es el intervalo real [−R, −(1 + r)];
- γ4 = γ4 (r) es el semicı́rculo de radio r, centrado en −1, que se encuentra en el semiplano
superior y orientado negativamente;
- γ5 = γ5 (r, R) es el intervalo real [−(1 − r), −r];
- γ6 = γ6 (r) es el semicı́rculo de radio r, centrado en 0, que se encuentra en el semiplano
superior y orientado negativamente;
y la curva cerrada CR,r := γ1 + γ2 + γ3 + γ4 + γ5 + γ6 . Ası́

f (z) dz = 0 .
CR,r

Además
iπ π2
Z !
Log(z)
lim f (z) dz = − i π Resz=−1 ( f ) = − i π = −iπ − = − .
r→0 γ4 (r) z − 1 z=−1
2 2
por el resultado en el Ejercicio 1 (nota el signo − debido a el cambio de orientación). Note que
log(r)2 + π2
Z Z p
πr

f (z) dz 6 | f (z)| | dz| 6
1 − r2

γ6 (r) γ6 (r)
y
log(R)2 + π2
Z Z p
πR ,

f (z) dz 6 | f (z)| | dz| 6
R2 − 1

γ2 (R) γ2 (R)
entonces
Z Z
log(R)
lim f (z) dz ∝ lim r| log(r)| = 0 , lim f (z) dz ∝ lim = 0.
r→0 γ6 (r) r→0 R→∞ γ2 (R) R→∞ R
Ası́
π2
Z
0 = lim f (z) dz = − + lim f (z) dz
R→+∞ CR,r 2 R→+∞ γ1 (r,R)+γ3 (r,R)+γ5 (r,R)
r→0 r→0
entonces
 

π2
Z Z Z
log(x)  
dx = lim f (z) dz = −  lim f (z) dz .
0 x2 − 1 R→+∞ γ1 (r,R) 2 R→+∞ γ3 (r,R)+γ5 (r,R)
r→0 r→0

Pero
−(1+r)
log(|x|) + i π −r
log(|x|) + i π
Z Z Z
f (z) dz = dx + dx
γ3 (r,R)+γ5 (r,R) −R x2 − 1 −(1−r) x2 − 1
3

entonces
0
log(|x|) + i π ∞ ∞
Z Z Z Z
log(x) 1
lim f (z) dz = dx = dx + i π dx .
R→+∞ γ3 (r,R)+γ5 (r,R) −∞ x2 − 1 0 x2 − 1 0 x2 −1
r→0
R∞
Como 0
1
x2 −1
dx = 0 se concluye

π2
Z
log(x)
2 dx = .
0 x2 − 1 2


Ejercicio 3
Demuestre que
+∞
π
Z
dx
PV = −√ .
−∞ x3 − 1 3

Solución.
Recuerde que
Z +∞ Z r−ε Z +∞ !
dx dx dx
PV := lim + , ε>0.
−∞ x3 − 1 r→0 −∞ x3 − 1 r+ε x3 − 1

Sea f (z) := 1
z3 −1
, entonces f dos polos simples en C+ := {z ∈ C | Imz > 0}, o sea z = 1 y z = e i 3 .
Note que

1 1
Res i 2π ( f ) = 2π
=  √  √
e 3 − i 3
(z − 1)(z − e ) z=e i π 3

− 23 + i 23 2 i 2
3


1 1 ei 3
=  √  =  √  = .
i 3 − 23 + i 21 3 − 21 − i 23 3

Sean R > r > 0 constantes positivas y consideremos la curva cerrada CR,r := γ1 + γ2 + γ3 + γ4 dada
por Sean y considere los arcos
- γ1 = γ1 (r, R) es el intervalo real [1 + r, R];
- γ2 = γ2 (R) es el semicı́rculo de radio R, centrado en el origen, que se encuentra en el semiplano
superior y orientado positivamente;
- γ3 = γ3 (r, R) es el intervalo real [−R, 1 − r];
- γ4 = γ4 (r) es el semicı́rculo de radio r, centrado en +1, que se encuentra en el semiplano
superior y orientado negativamente.
Por el teorema de los residuos (y R > 1)

π π
f (z) dz = 2π i Res 2π (f) = − √ − i .
CR,r ei 3 3 3
Note que
πR
Z Z

f (z) dz 6 | f (z)| | dz| 6 3
R −1

γ2 (R) γ2 (R)
4

o sea Z
lim f (z) dz = 0 .
R→+∞ γ2 (R)
De otro lado

iπ π
Z
lim f (z) dz = − i π Res1 ( f ) = − = −i
r→0 γ4 (R) + i 2π − i 2π
3
(z − e 3 )(z − e 3 ) z=1
por el resultado en el Ejercicio 1 (nota el signo − debido a el cambio de orientación). Entonces,
Z +∞
π π dx π
− √ − i = lim f (z) dz = PV 3−1
− i
3 3 R→+∞ CR,r −∞ x 3
r→0
o sea
+∞
π
Z
dx
PV = −√ .
−∞ x3 −1 3


Ejercicio 4
Sea γ : [0, 2π] → C definido por γ(θ) := e i θ + i e i 2θ . Calcule

1
I := 2015
dz .
γ z −1

Solución.
Si la función f (z) := 1
z2015 −1
no tiene polos en γ, entonces
X
I = f (z) dz = 2π i Reszk ( f )
γ zk

con zk los polos de f en el interior de γ. Veamos que f no tiene polos sobre γ u qué todos los polos
están en el interior de γ. Como

|γ(θ)|2 = e i θ + i e i 2θ e− i θ − i e− i 2θ = 2 − i e− i θ + i e i θ
 

= 2 + i 2 2 sin(θ) = 2 1 − sin(θ)


entonces |γ(θ)| = 1 si y sólo si sin(θ) = 1


2 o sea si y sólo si θ = π6 , 56 π. Pero es fácil ver che (e i θ )2015 , 1
si θ = π6 , 56 π. Ası́, f no tiene polos sobre γ.

Ahora veamos los polos dentro de γ. Del análisis anterior, se deduce que la raı́ces dentro de γ son
las que satisfacen
(  π "5 # )

R := e ∈ C θ ∈ 0, ∪ π, 2π ,
e i 2015θ
=1 .
6 6

La raı́ces de z2015 − 1, esatá dadas por z j := e i 2015 j , j = 0, 1, 2, . . . , 2014. Por lo que el problema se
reduce a encontrar j ∈ {0, 1, 2, . . . , 2014} tale que z j ∈ R o sea

j ∈ {0, 1, 2, . . . , 167} ∪ {840, 841, . . . , 2014} =: I .


5

Note que
2014
1 Y 1
f (z) = = ,
z2015 −1 j=0
z − zj
entonces
Y 1 1 1 zk
Reszk ( f ) = = d 2015
= 2014
= .
j=0
zk − z j dz (z − 1) z=z 2015(zk ) 2015
k
j,k

Por lo que  
   
2014
X  Y 1  2π i  X 
I = 2π i 
  =
  zk  .
 z k − z j  2015 
k∈I  j=0  k∈I
j,k

PONTIFICIA UNIVERSIDAD CATÓLICA DE CHILE
FACULTAD DE MATEMÁTICAS
DEPARTAMENTO DE MATEMÁTICAS

Clase de ejercicios 09 - 08/10/2015 ∗

Curso: Variable Compleja I


Sigla: MAT2705 - Segundo Semestre 2015
Profesor: Giuseppe De Nittis (gidenittis@mat.puc.cl)
Ayudante: Victor Cañulef Aguilar (vacanulef@uc.cl)

Ejercicio 1
Sea f : C → C entera tal que f (1) = z f (0). Demuestre que ∀ε > 0 existe z0 ∈ C tal que | f (z0 )| < ε.

Solución.
Si f ≡ 0 no hay nada que probar. Supongamos que f no es identicamente cero y que hay ε > 0 tal que
| f (z)| > ε para todos z ∈ C. Entonces f −1 es entera (ya que f (z) , 0 ∀z ∈ C). Además, | f −1 (z)| 6 ε,
por lo que es acotada, por lo que por el teorema de Liouville f −1 es constante. Ası́, f es constante y
como z f (0) = f (1) sigue que f ≡ 0, lo que es una contradicción. 

Ejercicio 2
Encuentre la serie de Laurent de las siguientes funciones:
(a)
1
f (z) := , en z = 1 y z = 0 ;
(z − 1)(z − 2)

(b)
1
f (z) := , en z = 1 y z = 0 .
z2 −z

Solución.
(a) Notemos que
+∞ +∞ !j
1 1 1 1 1 X  z j 1 X 1
f (z) = − = −   −   = − −
z−2 z−1 2 1 − 2z z 1 − 1z 2 j=0 2 z j=0 z

Entonces 
+∞ −( j+1)
 −2 si j > 0
X 

f (z) = aj zj ,

a j := 
 −1 si j < 0 .


j=−∞

∗Soluciones y notas estarán disponibles en la página web:


https://gdenittis.wordpress.com/courses/variable-compleja-i-mat2705/clase-de-ejercicios
1
2

Por otro lado


+∞
1 1 X
= − = − (z − 1)k
z−2 1 − (z − 1) j=0

ası́ que
+∞ +∞
1 X X
f (z) = − − (z − 1)k = (z − 1)k .
z − 1 j=0 j=−1

(b) Notemos que


+∞ +∞
1 1 1 X X
f (z) = − + = − − z = −
j
zj .
z z−1 z j=0 j=−1

Por otro lado


+∞ +∞
1 1 X 1 X
f (z) = − + = − (−1) j (z − 1) j + = − (−1) j+1 (z − 1) j .
−1 − (z − 1) z−1 j=0
z − 1 j=−1

Ejercicio 3
Sean f y g enteras tal que | f (z)| 6 |g(z)| para todos z ∈ C. ¿Que se puede concluir de f y g.

Solución.
f (z)
Suponemos que f no es identicamente cero. Sea h(z) := g(z) . Entonces |h(z)| 6 1 para todos z ∈ C.
Notemos que si z0 ∈ C es un cero de g, como h es acotada, la singularidad es removible. Ası́ h es
entera y acotada, por lo que es constante. Entonces, g serı́a multiplo de f . 

Ejercicio 4
Sea D := {z ∈ C | |z| 6 1}. Demuestre que no existe una función f : D → C no constante, analı́tica y
biyectiva con inversa analı́tica.

Solución.
Supongamos que existe tal f . Entonces, f −1 es entera y f −1 (C) = D implica | f −1 (z)| 6 1, es decir f −1
es acotada. Ası́ f −1 es una constante, lo que es una contadicción. 

Ejercicio 5 (Principio de reflexión de Schwarz)


Sea f = u + i v analı́tica en una región G + situada en ele semiplano superior, y que tiene un segmento
γ del eje real como parte de su frontera. Si f es continua y toma valores reales en γ, entonces f tiene
una continuación analı́tica única a través de γ hacia la regón G − , que es la reflexión de G + a través del
eje real.
3

Solución.
Defina
si z ∈ G + ∪ γ

 f (z)



F(z) := 
 f (z) si z ∈ G − .


Entonces, F es continua en G := G + ∪ γ ∪ G − . En z ∈ G − tenemos que
!
F(z) − F(z0 ) f (z) − f (z0 ) f (z) − f (z0 )
= = .
z − z0 z − z0 z − z0
Como z ∈ G + , l lı́mite existe luego F(z) es analı́tica en G − . Por tanto, F es una continuación analı́tica
de f en G . La unicidad es una consecuencia del Principio de identidad. 

Ejercicio 5 (Principio de identidad)


Supongamos que f y g son dos funciones analı́ticas en el abierto conexo U y que existe una sucesión
de puntos {zn } ⊂ U , convergente, de modo que f (zn ) = g(zn ), n = 1, 2, 3, . . .. Entonces, f = g en U .

Solución.
Sea z := limn zn . La función analı́tica F = f − g verifica F(zn ) = 0 para todos zn luego, por la
continuidad de F, F(z) = 0. Pero entonces z es un cero no aislado de F, con lo que F es constante, es
decir f = g en U . 
PONTIFICIA UNIVERSIDAD CATÓLICA DE CHILE
FACULTAD DE MATEMÁTICAS
DEPARTAMENTO DE MATEMÁTICAS

Clase de ejercicios 08 - 06/10/2015 ∗

Curso: Variable Compleja I


Sigla: MAT2705 - Segundo Semestre 2015
Profesor: Giuseppe De Nittis (gidenittis@mat.puc.cl)
Ayudante: Victor Cañulef Aguilar (vacanulef@uc.cl)

Ejercicio 1 (Fórmula de Wallis)


Obtenga la fórmula de Wallis
π
π (2n)!
Z
2
[cos(θ)]2n dθ = .
0 2 4n (n!)2

Solución.
Consideremos la función
1 2n
f (z) := z + z−1
z
y observemos que
22n
f e i θ := i θ [cos(θ)]2n .

e
Entonces

22n
Z
f (z) dz = i [cos(θ)]2n e i θ dθ
|z|=1 0 eiθ
Z 2π
= i 22n [cos(θ)]2n dθ
0
Z π
= i 22n 2 [cos(θ)]2n dθ
0
Z π
2
= i2 4 2n
[cos(θ)]2n dθ
0

Por otro lado


2n ! 2n ! !
1 X 2n 2n− j − j 1 X 2n 2(n− j) 1 2n
f (z) := z z = z = + Q(z)
z j=0 j z j=0 j z n


Soluciones y notas estarán disponibles en la página web:
https://gdenittis.wordpress.com/courses/variable-compleja-i-mat2705/clase-de-ejercicios
1
2

y
! !
1 2n 1 dz 1
f (z) dz = + Q(z) dz
2π i |z|=1 n 2π i |z|=1 z 2π i |z|=1
| {z }
=0
! !
2n 1 dz
=
n 2π i |z|=1 z
!
2n (2n)!
= = .
n (n!)2
Entonces
Z π
2 1
[cos(θ)]2n dθ = f (z) dz
0 i 22n 4 |z|=1
1 (2n)!
= 2n
2π i
i2 4 (n!)2
π 1 (2n)!
=
2 22n (n!)2
π (2n)!
= .
2 4n (n!)2


Ejercicio 2
La sucesió de Fibonacci Fn |n = 0, 1, 2, . . . N está definida inductivamente por

si n = 0




 0


si n = 1

Fn :=  1




 Fn−1 + Fn−2 si n > 2 .


Calcule el radio de convergencia R de


+∞
X
f (z) := Fn zn
n=0

y demuestre que f admite una extensión analı́tica en una vecindad de Re i θ para cada θ ∈ (0, 2π).

Solución.
Por el criterio del cociente de D’Alembert

1 Fn+1 F
= lim = lim n+1
R n→+∞ Fn n→+∞ Fn
si el lı́mite existe. Notemos que
Fn+1 Fn + Fn−1 Fn−1
= = 1 + , n>2.
Fn Fn Fn
Fn+1
Sea L := limn→+∞ Fn . Entonces la última ecuación implica

1
L = 1 + ⇒ L2 − L − 1 = 0
L
3

1+ 5
que tiene solución positiva L := 2 (número áureo). Entonces
1 2
R = = √ .
L 1+ 5
Ahora notemos que
+∞
X
f (z) = F n zn
n=1
+∞
X
= z + (Fn−1 + Fn−2 ) zn
n=2
X+∞ +∞
X
= z + z Fn−1 z n−1
+ z 2
Fn−2 zn−2
n=2 n=2

= z + z f (z) + z f (z) 2

lo que implica
z
f (z) = − . (0.1)
z2 +z−1

La (0.1) tiene una singularidad para z2 + z − 1 = 0, es decir si z±∗ := −1± 5
2 . Sin embargo la expresión
(0.1), definida en C − {z+∗ , z−∗ }, proporciona la extensión analı́tica de f en una vecindad de cualquier
punto z = Re i θ , θ ∈ (0, 2π). Por θ = 0 (o θ = 2π) el punto
√ √
2 2(1 − 5) 1− 5
z = R = √ = = = z+∗
1+ 5 1−5 −2
es (la unica) singularidad que se encuentra en el radio de convergencia. 

Ejercicio 3
Encuentre la serie de potencias de
1
f (z) =
z−1
en z0 = i .

Solución.
Notemos que
1 −1
=
z−1 1−z
−1
=
(1 − i ) − (z − i )
−1 1
=
(1 − i ) 1 − (z− i )
(1− i )
+∞ !k
−1 X z − i
=
(1 − i ) k=0 1 − i
+∞
X −1
= (z − i )k
k=0
(1 − i )k+1
√ √
y el radio de convergencia está dado por la condición |z − i | < |1 − i | = 2, ed decir R = 2. 
4

Ejercicio 4
Encuentre el radio de convergencia de la serie de potencias
z2 − 1
f (z) =
z3 − 1
en z0 = 2.

Solución.
Notemos que
z2 − 1 (z − 1)(z + 1)
=
3
z −1 (z − 1)(z2 + z + 1)
z+1
= 2
z +z+1

está definida para todos los puntos en C con exclusión de las raı́ces z2 +z+1 = 0, es decir z±+ := −1± i
2
3
.
Dado que
!2  √ 2
5  3  28
|2 − =
z±+ |2 +   = = 7
2 2 4
sigue que la distancia entre z0 = 2 y las singularidades de la función (es decir el radio de convergencia)

es R = 7. 

Ejercicio 5
Encuentre la expansión en serie de potencias de f (z) = Log(z) en z0 = i − 2. Muestre que su radio de

convergencia es 5 y explique por qué esto no contradice la discontinuidad de f en z = −2.

Solución.
¡Es importante recordar que la función log(z) se define en la superficie d Riemann R. Al costruir la

serie de Taylor es esencial especificar la rama de que se trata! Aquı́ se asume que z0 = i − 2 = 5e i 6 π
5

es un punto en la rama principal. Entonces en esta rama Log(z) = log(z).


De la regla de derivación para el logaritmo se obtiene que
dk f (−1)k+1 1 dk f 1 (−1)k+1
(z) = (k − 1)! ⇒ (z) = .
dzk zk k! dzk k zk
Entonces
+∞
X 1 (−1)k+1
f (z) = f ( i − 2) + z − ( i − 2) k .

k ( i − 2)k
k=1
El radio de convergencia se puede encontrar con el criterio de la raz de Cauchy. De hecho
! 1k
pk 1 1 1 1
lim |ak | = lim k
= √ lim √k = √ ,
k→+∞ k→+∞ k| i − 2| 5 k→+∞ k 5

entonces R = 5. Esto coincide con el hecho que z = 0 es una singularidad de Log(z) que se encuentra
sobre el crculo de convergencia.
Por otro lado, la función Log(z) es discontinua en el semieje negativo R− := (−∞, 0) y el punto −2 es
una discontinuidad interna al disco de convergencia dado que

−2 ∈ R− ∩ z ∈ C | |z − ( i − 2)| < 5 .

PONTIFICIA UNIVERSIDAD CATÓLICA DE CHILE
FACULTAD DE MATEMÁTICAS
DEPARTAMENTO DE MATEMÁTICAS

Clase de ejercicios 07 - 05/10/2015 ∗

Curso: Variable Compleja I


Sigla: MAT2705 - Segundo Semestre 2015
Profesor: Giuseppe De Nittis (gidenittis@mat.puc.cl)
Ayudante: Victor Cañulef Aguilar (vacanulef@uc.cl)

Ejercicio 1 (Lema de Jordan)


Sea z 7→ f (z) continua en la región Λσ := {z ∈ C | Im(z) > σ} con σ > 0 y suponga que limz→∞ f (z) =
0. Entonces, para todo número positivo t > 0, pruebe que
Z
lim e i zt f (z) dz = 0
R→+∞ ΓR

donde ΓR := {z ∈ C | |z| = R} ∩ Λσ .

Solución.
Note que
Z Z π−θ
∗ 

e f (z) dz =
i zt i tR cos(φ)+ i sin(φ)

e f (z(φ)) Re dφ
ΓR θ∗
Z π−θ∗
6 MΓR e−tR sin(φ) R dφ
θ∗
Z π
6 MΓR e−tR sin(φ) R dφ
0
Z π
2
= 2MΓR e−tR sin(φ) R dφ
0

donde MΓR := max{ f (z) | z ∈ ΓR } y θ∗ = θ∗ (R, σ) ∈ [0, π/2]. Dado que sin(φ) > π2 φ para φ ∈ [0, π/2]
(desigualdad de Kober-Jordan) encontramos que
Z Z π
2
e−tR π φ R dφ
2
i zt
e f (z) dz 6 2MΓR
ΓR 0
Z R
= πMΓR e−tα dα
0
πMΓR  
= 1 − e−tR .
t

Soluciones y notas estarán disponibles en la página web:
https://gdenittis.wordpress.com/courses/variable-compleja-i-mat2705/clase-de-ejercicios
1
2

Entonces,
πMΓR  π
Z

i zt
1 − e−tR = lim MΓR = 0.

lim e f (z) dz 6 lim
R→+∞ ΓR R→+∞ t t R→+∞

Ejercicio 2
Encuentre el radio de convergencia de las siguientes series:
(a) S := +∞ 3k
P
k=0 z .

(b) S := +∞ p
P
p=primo z .

(c) S := +∞ 3k zk
P
k=0 4k +5k .

(d) S := +∞ k! k
P
k=1 kk z .

(e) S := +∞
k
P k
k=3 [log(k)] z .
2

(f) S := +∞ z2k
P
k=1 (4k)k .

Solución.
(a) Notemos que
+∞
X +∞
X
S := z3k = aj zj
k=0 j=0
con
si j = 3n , n ∈ N ,

1



a j := 
0 en otro caso .


Ası́
1 q
:= lim sup j |a j | = lim sup a j = 1 ,
R j→+∞ j→+∞
entonces R = 1 por fórmula de Hadamard.

(b) Notemos que


+∞
X +∞
X
S := zp = aj zj
p=primo j=0
con
si j es primo ,

1



a j := 
0 en otro caso .


Ası́
1 q
:= lim sup j |a j | = lim sup a j = 1 ,
R j→+∞ j→+∞
entonces R = 1 por fórmula de Hadamard.
3k
(c) Dado que ak := 4k +5k
se tiene
! 1k
pk 3k 3 3 1
|ak | = = =  1k ,
4k + 5k (4k +
1
5k ) k 5  4  k
5 +1
3

Ası́
1 pk 3
:= lim |ak | = ,
R k→+∞ 5
entonces R = 5
3 por el criterio de la raı́z de Cauchy.
k!
(d) Dado que ak := kk
se tiene
!k
ak k! (k + 1)(k + 1)k (k + 1)k 1
= k = = 1 +
ak+1 k (k + 1)k! k k k
Entonces,
ak
lim = e,
k→+∞ ak+1
es decir R = e por el criterio del cociente de D’Alembert.
k
(e) Dado que ak := [log(k)] 2 se tiene
 1
pk k k 1
|ak | = log(k) 2 = [log(k)] 2

lo cual implica
1 pk
:= lim |ak | = +∞
R k→+∞
y R = 0 por el criterio de la raı́z de Cauchy.

(f) Notemos que


+∞ +∞
X z2k X
S := k
= aj zj
k=1
(4k) j=0
con
 1
si j = 2k, k ∈ N ∪ {0} ,



 (4k)k

a j := 

si j = 2k + 1, k ∈ N ∪ {0} .

0

Ası́
! 2k1 r
1 q 1 1
:= lim sup j |a j | = lim sup k
= lim = 0,
R j→+∞ k→+∞ (4k) k→+∞ 4k
entonces R = +∞ por fórmula de Hadamard. 

Ejercicio 3
¿Que funciones son representadas por las siguientes series de potencias?
(a) S 1 := +∞ k
P
k=1 k z ,
P+∞ 2 zk .
(b) S 2 := k=1 k

Solución.
(a) Notemos que
+∞
 +∞   +∞ 
X X  X dzk 
S1 = k zk = z  k zk−1  = z  
k=1 k=1 k=1
dz 
4

y por el teorema de Weierstrass


 +∞   +∞  !
d X k  d X k  d 1 z
S1 = z  z  = z  z − 1 = z − 1 = , |z| < 1 .
dz k=1 dz k=0 dz 1 − z (1 − z)2
Notemos que |z| < 1 = R es el cı́rculo de convergencia de S 1 .

(b) Notemos que


+∞
 +∞   +∞ 
X X  X dzk 
S2 = k2 zk = z  k2 zk−1  = z  k 
k=1 k=1 k=1
dz 
y por el teorema de Weierstrass
 +∞ 
1+z
!
d X k  dS 1 d z
S2 = z  k z  = z = z 2
= z , |z| < 1 .
dz k=1 dz dz (1 − z) (1 − z)3
Notemos que |z| < 1 = R es tambien el cı́rculo de convergencia de S 2 . 

Ejercicio 4
Demuestre que para cada θ ∈ R, no existe ninguna vecindad de z = e i θ en C donde la función
+∞
X
f (z) := zk!
k=1
sea analı́tica.

Solución.
Notemos que si p y q son coprimos y k > q, entonces
 p
k!  pq  k!q
r e i q 2π = rk! e i q 2π = rk!
k!
dado que q ∈ N. Por lo tanto,
q−1 +∞ q−1 +∞
i qp 2πk! p
X X X X
f (z) = z k!
+ z k!
= k!
r e + rk! , si z = r e i q 2π .
k=1 k=q k=1 k=q

Ası́

+∞ +∞ +∞ +∞
q−1
q−1 q−1
p p
X X X X X X X
| f (z)| = rk! e i q 2πk! + rk! − rk! e i q 2πk! >

rk! > rk! − rk! > rk! − (q−1)

k=1 k=q k=q k=1 k=q k=1 k=q

para r < 1. Por lo que | f (z)| → +∞ si hacemos r → 1. De lo que se deduce que f no tiene extensión
p
analitica en e i q 2π . Entonces, f no se extiende analı́ticamente en e i 2πθ si θ ∈ Q ∩ [0, 1]. Como
Θ := {e i 2πθ ∈ C | θ ∈ Q ∩ [0, 1]} es denso en C1 := {z ∈ C | |z| = 1}, se concluye el resultado. 
PONTIFICIA UNIVERSIDAD CATÓLICA DE CHILE
FACULTAD DE MATEMÁTICAS
DEPARTAMENTO DE MATEMÁTICAS

Clase de ejercicios 06 - 21/09/2015 ∗

Curso: Variable Compleja I


Sigla: MAT2705 - Segundo Semestre 2015
Profesor: Giuseppe De Nittis (gidenittis@mat.puc.cl)
Ayudante: Victor Cañulef Aguilar (vacanulef@uc.cl)

Ejercicio 1 (Polinomios de Laguerre)


Los polinomios de Laguerre Ln , n ∈ N, está dados por
1 z dn n −z 
Ln (z) := e z e .
n! dzn
(a) Muestre que

ez ξn e−ξ
Ln (z) := dξ
2π i γ (ξ − z)n+1
donde z está en el interior de la curva de Jordan SPP γ.

(b) La función generatriz de los polinomios de Laguerre viene dada por


+∞
 N 
X X 
G(t, z) = t Ln (z) := lim  t Ln (z) .
n  n
N→+∞
n=0 n=0
Muestre que
1 t
G(t, z) = e−z 1−t , z∈C
1−t
para |t| suficientemente pequeño†.

Solución.
(a) Basta usar la fórmula de Cauchy para derivadas:
dn n −z 
n! e−z Ln (z) = z e
dzn

n! ξn e−ξ
= dξ .
2π i γ (ξ − z)n+1
Entonces,
ez ξn e−ξ
Ln (z) = dξ .
2π i γ (ξ − z)n+1
(b) Note que

Soluciones y notas estarán disponibles en la página web:
https://gdenittis.wordpress.com/courses/variable-compleja-i-mat2705/clase-de-ejercicios

Por prolongación analtica se puede mostrar que la formula es valida para todo |t| < 1
1
2

N  N !n 
X ez e−ξ X tξ
tn Ln (z) =

 dξ
ξ − z n=0 ξ − z 

n=0
2π i γ

Sea [0, 2π] 3 φ 7→ ξ(φ) ∈ γ la parametrización dada por ξ(φ) := z + re i φ , 0 < r < |z|. A lo largo de la
curva

tξ = |t| |z + re | = |t| z + e i φ < 1

ξ − z ∀ φ ∈ [0, 2π]
r r

si |t| se elige suficientemente pequeño (note que r puede ser arbitrariamente grande). Entonces, la serie
geométrica converge uniformemente

∞ !n
X tξ 1 ξ−z
= =
ξ−z 1− tξ ξ − z − tξ
n=0 ξ−z

y
+∞
X ez e−ξ
t Ln (z) =
n

n=0
2π i γ ξ − z − tξ

ez e−ξ
= z dξ .
2π i (1 − t) γ ξ − 1−t

De la fórmula integral de Cauchy se obtiene

+∞
X ez z 1 t
tn Ln (z) = e− 1−t = e−z 1−t
n=0
(1 − t) (1 − t)

es decir, el resultado. 

Ejercicio 2
Sea z 7→ f (z) entera tal que z 7→ z−n f (z) es acotada si |z| > R. Demuestre que f es un polinomio de
grado a lo más n.

Solución.
Notemos que si hay un M > 0 tal que


f (z) 6 M si |z| > R
zn

tenemos que

dn+1 f (n + 1)! f (w)
n+1
(z) = dw , |z| ≤ R0 .
dz 2π i |w|=R0 (w − z)n+2
3

R0
Sea R0 > R y |z| 6 2. Entonces,


(n + 1)!
n+1
d f | f (w)|
n+1 (z) 6 n+2
| dw|
dz 2π |w|=R0 |w − z|

(n + 1)!M 1
6 | dw|
2π |w|=R0 (|w| − |z|)n+2

(n + 1)!M 1
= 0 n+2
| dw|
2π |w|=R0 (R − |z|)

(n + 1)!M 2n+2
6 | dw|
2π 0 n+2
|w|=R0 R
(n + 1)!M 2n+2
= n+2
2πR0
2π R 0
(n + 1)!M n+2
= 2 .
R0 n+1

Por lo que al hacer R0 → +∞, se obtiene que f (n+1) ≡ 0. Ası́ f (m) ≡ 0 si m > n + 1. Entonces, f tiene
a lo más n + 1 términos no nulos en su serie de potencias, de lo que se deduce el resultado. 

Ejercicio 3
Sea A ⊆ C una región simplemente conexa y h : [a, b] × A → C tal que h es continua y z 7→ h(t0 , z)
es analı́tica para t0 ∈ [a, b] fijo. Muestre que
Z b
H(z) := h(t, z) dt , z∈A
a

es analı́tica en A .

Solución.
Primero veamos que H es continua. Sea z0 ∈ A y Dε (z0 ) := {z ∈ C | |z0 −z| 6 ε}. Para ε suficientemente
pequeño Dε (z0 ) ⊂ A (A es un abierto) y h es uniformemente continua sobre el compacto [a, b] ×
Dε (z0 ). Entonces, si z ∈ Dε (z0 ),

Z b
lim H(z) − H(z0 ) 6 h(t, z) − h(t, z ) dt
0
z→z0 a
Z b
6 C |z − z0 | dt
a
= C (b − a) |z − z0 |

es decir la continuidad de H en z0 . Ahora sea R ⊂ A un rectangulo con los lados paralelos a los ejes
x = Re(z) y y = Im(z). Luego, por el lema de Goursat:

h(t0 , z) dz = 0, para t0 ∈ [a, b] fijo .
∂R
4

Entonces,
Z b !
0 = h(t, z) dz dt
a ∂R
Z b !
= h(t, z) dt dz
∂R a

= H(z) dz
∂R

dado que las condiciones del teorema de Fubini se cumplen. Sigue que, γ
H(z) dz = 0 a lo largo de
cada curva cerrada SPP γ ⊂ A (para aproximación de γ con una colección de ∂R j , j = 1, 2, . . .).
Finalmente, del teorema de Morera, se concluye el resultado. 

Ejercicio 4
Sea g : [a, b] → C continua. Muestre que
Z b
H(z) := g(t) e− i tz dt
a

es entera y satisface |H(z)| 6 CeA|Im(z)| con A, C > 0.

Solución.
Sea h : [a, b] × C → C dada por h(t, z) := g(t) e− i tz . Entonces, h es continua y z 7→ h(t0 , z) es analı́tica
para t0 ∈ [a, b] fijo. Por el problema anterior, H es entera. Además:
Z b
|H(z)| 6 |g(t) e− i tz | dt
a
Z b
0
6 C |e− i t Re(z) et Im(z) | dt
a
Z b
= C 0
et Im(z) dt
a
6 C 0 (b − a) eA |Im(z)|
donde C 0 := maxt∈[a,b] {g(t)} y A := max{b, 0} si Im(z) > 0 o A := max{−a, 0} si Im(z) < 0. El resultado
se obtiene al poner C := C 0 (b − a). 
PONTIFICIA UNIVERSIDAD CATÓLICA DE CHILE
FACULTAD DE MATEMÁTICAS
DEPARTAMENTO DE MATEMÁTICAS

Clase de ejercicios 05 - 15/09/2015 ∗

Curso: Variable Compleja I


Sigla: MAT2705 - Segundo Semestre 2015
Profesor: Giuseppe De Nittis (gidenittis@mat.puc.cl)
Ayudante: Victor Cañulef Aguilar (vacanulef@uc.cl)

Ejercicio 1
Si z 7→ f (z) es analı́tica y acotada por M > 0, prueba que
(n) MRn!
f (z) 6 , |z| < R .
(R − |z|)n+1

Solución.
Si aplicamos la fórmula de Cauchy para derivadas al dominio DR := {z ∈ C | |z| < R}, obtenemos

n! f (w) n! f (w)
f (z) =
(n)
n+1
dw = dw , z ∈ DR .
2π i ∂DR (w − z) 2π i |w|=R (w − z)n+1
Por lo que
(n) n! f (w)
f (z) =

n+1
dw
2π |w|=R (w − z)


n! | f (w)|
6 | dw|
2π |w|=R |w − z|n+1

n!M 1
6 | dw|
2π |w|=R |w| − |z|n+1

n!M 1
= | dw|
2π R − |z|n+1 |w|=R
MRn!
=
R − |z| n+1


donde hemos utilizado ||w| − |z|| 6 |z − w| y |z| < |w| = R. 

Ejercicio 2 (Polinomio de Legendre)


El polinomio de Legendre Pn , n ∈ N, se define como
1 dn  2
(z − 1)n .

Pn (z) := n n
2 n! dz

Soluciones y notas estarán disponibles en la página web:
https://gdenittis.wordpress.com/courses/variable-compleja-i-mat2705/clase-de-ejercicios
1
2

Muestre que

1 (ξ2 − 1)n
Pn (z) := dξ
2π i γ 2n (ξ − z)n+1
donde z está en el interior de la curva de Jordan SPP γ.

Solución.
Basta usar la fórmula de Cauchy para derivadas:
dn  2
2n n! Pn (z) = (z − 1)n

dz n

n! (ξ2 − 1)n
= dξ .
2π i γ (ξ − z)n+1
Entonces,
1 (ξ2 − 1)n
Pn (z) = dξ .
2π i γ 2n (ξ − z)n+1


Ejercicio 3
Sea z 7→ P(z) un polinomio, ninguna de cuyas raı́ces se encuentra sobre la curva de Jordan SPP γ.
Muestre que
1 1 dP
N = (z) dz
2π i γ P(z) dz
donde N ∈ N es el número de raı́ces de P en el interior de γ (incluyendo multiplicidades).

Solución.
Por el teorema fundamental del álgebra, todo polinomio de grado n se puede escribir de la forma

P(z) = a (z − z1 ) (z − z2 ) . . . (z − zn ) , a, z1 , z2 , . . . zn ∈ C .

Como consecuencia de la regla de Leibniz


 
n 

 n

dP X Y 
(z) = a  (z − zk ) .
dz j=1 

 k=1


k, j

Entonces  
Pn Qn 
a j=1  k=1 (z − zk )

1 dP k, j
(z) =
P(z) dz a (z − z1 ) (z − z2 ) . . . (z − zn )
1 1 1
= + + ... +
(z − z1 ) (z − z1 ) (z − zn )
ası́
n
1 1 dP X 1 1
(z) dz = dz .
2π i γ P(z) dz j=1
2π i γ (z − z j )
La prueba es completada mediante la fórmula de Cauchy que dice

 1 si z j es en el interior de γ

1 1


dz = 

2π i γ (z − z j )  0 si z j no es en el interior de γ .


3

Ejercicio 4 (Lema de Schwarz)

f (z)
(a) Sea z 7→ f (z) analı́tica en D := {z ∈ C | |z| < 1} y que satisface f (0) = 0. Defina F(z) := z
para todo z en 0 < |z| < 1. ¿Qué valor puede dársele a F(0) para que F sea analı́tica en |z| < 1?

(b) Con base en el principio del máximo pruebe el Lema de Schwarz: Sea z 7→ f (z) como en (a) y
que satisface | f (z)| 6 1. Entonces | f (z)| 6 |z| y | f 0 (0)| 6 1, con la igualdad que se cumple sólo
si f (z) = e i φ z para φ ∈ R fijo.

(c) Además muestre que | f (z)| 6 1 para |z| < 1 en (b) implica que | f 0 (0)| 6 1 independientemente
del valor de f (0).

Solución.
(a) Para que F sea analı́tica, al meno deba ser continua, por lo que definimos
f (z) f (z) − f (0) df
F(0) := lim := lim = (0) .
z→0 z z→0 z−0 dz
Para verificar que F es analı́tica en z = 0 observamos que para la fórmula de Cauchy
!
F(h) − F(0) 1 f (h) d f
= − (0)
h h h dz
!
1 1 f (z) f (z)
= dz − 2
dz
h 2π i |z|=r<1 z(z − h) |z|=r<1 z

1 f (z)
= 2
dz .
2π i |z|=r<1 z (z − h)
Sea
1 f (z)
G := dz
2π i |z|=r<1 z3
es un número bien definido tal que |G| 6 donde M := max{| f (z)| | |z| = r < 1}. Entonces, si |h| < r
M
r2

F(h) − F(0) − G = 1 h f (z)
dz

h 3
2π |z|=r<1 z (z − h)

M 1
6 |h| 3
| dz|
2πr |z|=r<1 |z − h|

M 1
6 |h| 3
| dz|
2πr |z|=r<1 |z| − |h|
M 2πr
6 |h|
2πr3 r − |h|
|h| M
=
r − |h| r2
lo que implica
dF F(h) − F(0)
(0) = lim = G
dz h→z h
es decir la analiticidad de F en z = 0.
4

(b) F es analı́tica en Dr := {z ∈ C | |z| < r} para todo r 6 1. Por el principio del máximo, la función
F, restringida en el conjunto cerrado Dr , alcanza su máximo en la fronteira ∂Dr := {z ∈ C | |z| = r}.
Entonces, para todo r 6 1, hay un zr ∈ ∂Dr tal que

f (zr ) | f (zr )| 1
|F(z)| 6 |F(zr )| = = 6 , z ∈ Dr .
zr r r
Como r → 1 obtenemos |F(z)| 6 1 para todo z ∈ D, es dicir

| f (z)| 6 |z| , z∈D .

En particular en z = 0
d f (0) = |F(0)| 6 1 .
dz
Por otro lado, si existe a z∗ ∈ D tal que | f (z∗ )| = |z∗ | o | f 0 (0)| = 1, entonces F(z∗ ) = 1 alcanzarı́a su
máximo en D, por lo que serı́a constante, y ası́ F(z) = λ, es dicir f (z) = λz con |λ| = 1.

(c) Para eso basta usar la formula de Cauchy:



d f (0) = 1 f (z)
dz
dz 2π |z|=r<1 z2


1 | f (z)|
6 | dz|
2π |z|=r<1 r2

1 1
6 | dz|
2π r2 |z|=r<1
1
=
r
para todo 0 < r 6 1. Entonces, para r = 1 sigue el resultado. 
PONTIFICIA UNIVERSIDAD CATÓLICA DE CHILE
FACULTAD DE MATEMÁTICAS
DEPARTAMENTO DE MATEMÁTICAS

Clase de ejercicios 02 - 24/08/2015


Curso: Variable Compleja I
Sigla: MAT2705 - Segundo Semestre 2015

Profesor: Giuseppe De Nittis. (gidenittis@mat.puc.cl)


Ayudante: Victor Cañulef Aguilar. (vacanulef@uc.cl)

Pagina Web: https://gdenittis.wordpress.com/courses/variable-compleja-i-mat2705/clase-de-ejercicios/

Ejercicio 1

Sea h : [0, 1] → C continua. Sea H : C − [0, 1] → C tal que


Z 1
h(t)
H(z) := dt .
0 t−z
Demuestre que H es analı́tica en C − [0, 1].

Solución.
Sea z ∈ C − [0, 1] y {hn }n∈N tal que hn → 0 si n → +∞. Notemos que
H(z + hn ) − H(z)
Z 1 !
1 1 1
= dt h(t) −
hn hn 0 t − z − hn t−z
Z 1 !
1 hn
= dt h(t)
hn 0 (t − z − hn )(t − z)
Z 1
h(t)
= dt .
0 (t − z − hn )(t − z)
h(t) h(t)
La secuencia (t−z−hn )(t−z) converge uniformemente hasta el lı́mite (t−z)2
. Para ver lo anterior, basta notar
que

h(t) − h(t) = |h(t)| −hn 6 |h(t)| |hn |
(t − z)2 (t − z − hn )(t − z) |t − z| (t − z − hn )(t − z) |t − z|2 |t − z| − |hn |
lo que implica
!
lim
h(t)

h(t) = |h(t)| lim 1
lim |hn | = 0 .
n→∞ (t − z)2 (t − z − hn )(t − z) |t − z|2 n→∞ |t − z| − |hn | n→∞
Por esta razón se puede intercambiar el lı́mite con la integral y
H(z + hn ) − H(z)
Z 1
h(t)
0
H (z) := lim = dt .
n→∞ hn 0 (t − z)2

1
2

Ejercicio 2

Sea f analı́tica en una región A. Sea


g(z) := f z .


Demuestre que g es analı́tica en A∗ := {z ∈ C | t. q. z ∈ A}.

Solución.
Sea z ∈ A∗ . Notemos que z ∈ A y

g(z + h) − g(z) f z+h − f z  f z + h − f z 


    
= =   .
h h h
Entonces
g(z + h) − g(z)  f z + h − f z 
  
= lim   = f 0 z

lim
h→0 h h→0 h
ya que z 7→ z es continua en C. 

Ejercicio 3: Forma polar de las ecuaciones de Cauchy-Riemann

(a) Deducir la forma polar de las ecuaciones de Cauchy-Riemann.

(b) Prueba que la función


 
f (z) = zm = rm cos(mθ) + i sin(mθ) , m∈N

satisface las ecuaciones de Cauchy-Riemann en forma polar para todo z , 0.

Solución.
(a) Sean z = re i θ y f (z) = u(r, θ) + i v(r, θ). Notemos que
∂u ∂u ∂x ∂u ∂y ∂u ∂u
= + = cos θ + sin θ
∂r ∂x ∂r ∂y ∂r ∂x ∂y
(0.1)
∂u ∂u ∂x ∂u ∂y ∂u ∂u
= + = − r sin θ + r cos θ
∂θ ∂x ∂θ ∂y ∂θ ∂x ∂y
y análogamente
∂v ∂v ∂x ∂v ∂y ∂v ∂v
= + = cos θ + sin θ
∂r ∂x ∂r ∂y ∂r ∂x ∂y
(0.2)
∂v ∂v ∂x ∂v ∂y ∂v ∂v
= + = − r sin θ + r cos θ
∂θ ∂x ∂θ ∂y ∂θ ∂x ∂y
ya que x(r, θ) = r cos θ y y(r, θ) = r sin θ. Si las derivadas parciales con respecto a x y y satisfacen las
ecuaciones de Cauchy-Riemann, las ecuaciones (0.2) pasan a ser
∂v ∂u ∂u
= − cos θ + sin θ
∂r ∂y ∂x
(0.3)
∂v ∂u ∂u
= r sin θ + cos θ .
∂θ ∂y ∂x
3

De las ecuaciones (0.1) y (0.3) se deduce que la forma polar de las ecuaciones de Cauchy-Riemann

∂u 1 ∂v 1 ∂u ∂v
= , = − .
∂r r ∂θ r ∂θ ∂r

(b) Como
u(r, θ) := rm cos(mθ) , v(r, θ) := rm sin(mθ)
resulta que
∂u ∂u
= m rm−1 cos(mθ) , = −m rm sin(mθ)
∂r ∂θ
∂v ∂v
= m rm−1 sin(mθ) , = m rm cos(mθ) .
∂r ∂θ


Ejercicio 4
Para mostrar que las las ecuaciones de Cauchy-Riemann no son suficientes para la analiticidad se
considera la siguiente función
p
f (z) = |Re(z) Im(z)| .
(a) Prueba que f es continua en z = 0.

(b) Prueba que las ecuaciones de Cauchy-Riemann se cumplen en z = 0.

(c) Prueba que f no tiene derivada en z = 0.

Solución.
(a) De la desigualdad 0 6 (|Re(z)| − |Im(z)|)2 = |Re(z)|2 + |Im(z)|2 − 2|Re(z) Im(z)| se deduce que
1 p |z|
| f (z)| 6 √ |Re(z)|2 + |Im(z)|2 = √ .
2 2
Entonces, f es continua en z = 0 y f (0) = 0.
∂v ∂u
(b) Como f (z) ∈ R, v(x, y) = 0 y = = 0. Como u(x, y) =
p
∂x ∂x |xy|
∂u u(, 0) − u(0, 0) ∂u u(0, ) − u(0, 0)
(0, 0) = lim = 0, (0, 0) = lim = 0.
∂x →0  ∂y →0 
Entonces, las ecuaciones de Cauchy-Riemann se cumplen en z = 0.

(c) Notemos que


p
f (z) − f (0) f (z) |xy|
= = .
z z x + iy
Si x(t) = tx0 y y(t) = ty0 con x0 y y0 constantes se deduce que
p p
f (z(t)) − f (0) t2 |x0 y0 | |x0 y0 |
lim = = .
t→0 z(t) t(x0 + i y0 ) x0 + i y0
Entonces, el lı́mite no es único y la función no tiene derivada en z = 0.

4

Ejercicio 5

(a) Hallar todas las raı́ces de la ecuación

sin z = cosh 4 .

(b) Hallar todas las raı́ces de la ecuación

cos z = 2 .

Solución.
(a) A partir de la identidad
e i z − e− i z
sin z = = sin x cosh y + i cos x sinh y
2i
se deduce que
sin x cosh y = cosh 4 , cos x sinh y = 0 (0.4)
Como sinh y = 0 implica y = 0 la segunda ecuación tiene soluciones
 π 
A := (x ∈ R , y = 0) ; y B := x = + kπ , y ∈ R − {0} , k ∈ Z .
2
Como cosh y = 1 en los puntos de tipo A la primera ecuación (0.4) resulta sin x = cosh 4 > 1 que no
tiene ninguna solución. En los puntos de tipo B resulta que sin( π2 + kπ) = (−1)k . Como cosh y > 1 la
primera ecuación (0.4) puede cumplirse sólo para k par. Entonces, todas las raı́ces son
π 
zk = + 2kπ + i 4 , k∈Z.
2
(b) A partir de la identidad
e i z + e− i z
cos z = = cos x cosh y − i sin x sinh y
2
se deduce que
cos x cosh y = 2 , sin x sinh y = 0 (0.5)
Como sinh y = 0 implica y = 0 la segunda ecuación tiene soluciones

A := (x ∈ R , y = 0) ; y B := (x = kπ , y ∈ R − {0}) , k∈Z.

Como cosh y = 1 en los puntos de tipo A la primera ecuación (0.5) resulta cos x = 2 > 1 que no tiene
ninguna solución. Een los puntos de tipo B resulta que cos(kπ) = (−1)k . Como cosh y > 1 la primera
ecuación (0.5) puede cumplirse sólo para k par. Entonces, todas las raı́ces son

zk = 2kπ + i y0 , k∈Z.

con cosh y0 = 2. Esta ecuación es equivalente a (ey0 )2 − 4ey0 + 1 = 0 y tiene la soluciones ey0 = 2 ± 3.
Por lo tanto

zk = 2kπ + i log(2 ± 3) , k∈Z.

PONTIFICIA UNIVERSIDAD CATÓLICA DE CHILE
FACULTAD DE MATEMÁTICAS
DEPARTAMENTO DE MATEMÁTICAS

Clase de ejercicios 04 - 07/09/2015 ∗

Curso: Variable Compleja I


Sigla: MAT2705 - Segundo Semestre 2015
Profesor: Giuseppe De Nittis (gidenittis@mat.puc.cl)
Ayudante: Victor Cañulef Aguilar (vacanulef@uc.cl)

Ejercicio 1 (Una aplicación del teorema de Green)


Sea A un dominio acotado con frontera suave. Muestre que:

1
z dz = Area(A ) .
i 2 ∂A

Solución.
Elija una parametrizació [a, b] 3 t 7→ z(t) para ∂A . Un cálculo directo muestra:
Z b !
dx dx
z dz = (x(t) − i y(t)) (t) + i (t) dt
∂A a dt dt
Z b ! Z b !
dx dy dy dx
= x(t) (t) + y(t) (t) dt + i x(t) (t) − y(t) (t) dt
a dt dt a dt dt
! !
= y dy − (−x) dx + i x dy − y dx .
∂A ∂A

Aplicando el teorema de Green resulta que


" "
∂y ∂(−x) ∂x ∂y
! !
z dz = + dx dy + i + dx dy
∂A A ∂x ∂y A ∂x ∂y
"
= 0 + i2 dx dy
A
= i 2 Area(A ) .


Ejercicio 2 (Fórmula integral de Cauchy)


Evalue los siguientes integrales con la ayuda de la fórmula integral de Cauchy.


Soluciones y notas estarán disponibles en la página web:
https://gdenittis.wordpress.com/courses/variable-compleja-i-mat2705/clase-de-ejercicios
1
2

(a)
zm
dz , m ∈ N ∪ {0};
|z|=2 z−1

(b)
zm
dz , m ∈ N ∪ {0};
|z|=1 z−2

(c)
ez
dz , m = 0, 1;
|z|=1 zm

(d)
ez
dz .
|z|=1 z(z − 1)

Solución.
(a) Recordemos la fórmula integral de Cauchy

zm
2π i w m
= dz
∂A z−w
con A ⊂ C tal que w ∈ A (zm con m > 0 es analı́tica en todo C). Dado que 1 ∈ D2 := {z ∈ C | |z| 6 2}
y ∂D2 := {z ∈ C | |z| = 2} sigue que

zm
dz = 2π i (1)m = 2π i .
|z|=2 z−1

zm
(b) Notemos que z−2 es analı́tica en |z| > 1. Por lo que

zm
dz = 0
|z|=1 z − 2

por la fórmula integral de Cauchy.


dez
(c) La función ez es analı́tica en C y verifica ez = dz . Entonces, el teorema de Cauchy(-Goursta)
establece que
ez dz = 0 .
|z|=1
Por m = 1, la fórmula integral de Cauchy establece que

ez ez
dz = dz = 2π i e0 = 2π i .
|z|=1 z |z|=1 z − 0

(d)
ez ez ez
!  
dz = − dz = 2π i e1 − e0 = 2π i (e − 1) .
|z|=1 z(z − 1) |z|=1 z−1 z


Ejercicio 3 (Integrales de Fresnel)


Prueba las igualdades
+∞ +∞
π
Z Z r
cos(t ) dt =
2
sin(t ) dt =
2
.
0 0 8
3

Solución.
2
Al aplicar la fórmula integral de Cauchy a la función f (z) = e−z a lo largo de la frontera del sector
 π
D := z ∈ C | 0 6 z 6 R, 0 6 Arg(z) 6
8

se obtiene
2
2 (z − a) e−z 2
e−z dz = dz = [(z − a)e−z ]z=0 = 0 , z ∈ D.
∂D ∂D z−a

Pero, ∂D = γ1 + γ2 + γ3 , donde

γ1 : [0, R] 3 t 7→ z(t) := t
 π
γ2 : 0, 3 t 7→ z(t) := Re i t
4
π
γ3 : [0, R] 3 t 7→ z(t) := (R − t) e i 4

Entonces,
Z R Z π Z R π
2 2 4 2 e i 2t π 2e i 2
0= e−z dz = e−t dt + i R e−R e i t dt − e i 4 e−(R−t) dt
∂D 0 0 0

es decir
π
Z R Z R Z
π 4
− i (R−t)2 −t2 2 e i 2t
e i 4 e dt = e dt + i R e−R e i t dt .
0 0 0

Note que
Z R Z R Z R
π π π
− i (R−t)2 − i (R−t)2 2
e i 4 e dt = −e i 4 e d(R − t) = e i 4 e− i t dt
0 0 0

y


π
Z
−t2
e dt =
0 2
dado que es la mitad de la integral de Gauss. Sigue que
√ Z π
∞  π
Z 
π 4
− i t2 2 e i 2t
dt = e −i
+ i lim R −R
e dt .
i t 
e 4  e
0 2 R→∞ 0

Note que
Z π Z π
4 4
2 i 2t
2 i 2t
−R e it e−R e dt
R e e dt 6 R
0 0
Z π
4 2
= R e−R cos(2t)
dt
0
Z π
R 2 2
= e−R cos(t)
dt
2 0
4

2 2 (1− π t) π
y dado que cos(t) > 1 − π2 t (la desigualdad de Kober†) implica e−R cos(t) 6 e−R 2 para 0 6 t 6 2 se
obtiene Z π Z π
4
−R2 e i 2t i t R 2 −R2 (1− 2 t)
R e e dt 6 e π dt
2 0

0

π
Z 0
= e s ds
4R −R2
π 2
= (1 − e−R ) .
4R
Por lo tanto Z π
4 2 i 2t
lim R e−R e e i t dt = 0
R→∞ 0
y √ ! √

π π
Z
− i t2 1 1 π
e dt = e = √ + i √ −i 4 .
0 2 2 2 2
R∞ 2
R ∞  R ∞ 
Para concluir la prueba basta notar que 0 e− i t dt = 0 cos(t2 ) dt + i 0 sin(t2 ) dt . 


A partir de
 t 2 t t
1 − cos(t) = 2 sin y 2 sin cos = sin(t)
2 2 2
π
y utilizando sin(t) 6 1 para 0 6 t 6 2
se obtiene que
 2
t
2 sin 2
t π
1 − cos(t) 6 = tan , 06t6 .
sin(t) 2 2
Note que tan(x) 6 4
π
x dado que tan(x) = 0, tan( π4 ) = 1 y la función x 7→ tan(x) es convexa para 0 6 x 6 π4 . La sustitución
x= t
2
concluye la prueba de la desigualdad de Kober.
PONTIFICIA UNIVERSIDAD CATÓLICA DE CHILE
FACULTAD DE MATEMÁTICAS
DEPARTAMENTO DE MATEMÁTICAS

Clase de ejercicios 03 - 31/08/2015


Curso: Variable Compleja I
Sigla: MAT2705 - Segundo Semestre 2015

Profesor: Giuseppe De Nittis. (gidenittis@mat.puc.cl)


Ayudante: Victor Cañulef Aguilar. (vacanulef@uc.cl)

Pagina Web: https://gdenittis.wordpress.com/courses/variable-compleja-i-mat2705/clase-de-ejercicios/

Ejercicio 1 (Valor principal de log z)


El logaritmo log : R → C es el mapeo inverso de la exponencial con dominio de definición R dado da
una superficie de Riemann1. Llamamos a la rama de R cortada a lo largo del eje real negativo, que se
mapea en la franja semiinfinita −π 6 Im(z) < +π, rama principal del logaritmo. Denotamos la función

Figure 0.1. Rama principa de R.

logaritmo cuando se restringe la rama principal, por

Log z := log |z| + i Arg(z)

donde Arg(z) es el ergumento principal de z. La función Log z se llama valor principal de log z.
(a) Prueba que Log i = i π2 .

1R consiste en un número infinito de copias de C − {0} apiladas en capas unas sobra otras, cada una cortada a lo largo del

eje real negativo con el borde superior de una capa “pegada” al borde inferior de la capa superior.
1
2

(b) Prueba que Log (−1 + i ) = log 2 + i π 34 .

(c) Prueba que Log [ i (−1 + i )] = log 2 − i π 43 .

(d) Se concluya que las propiedades usuales de lo logaritmo pueden non cumplirse por Log z.

Solución.
π
(a) Log i = log | i | + i Arg( i ) = log 1 + i Arg( i ) = i Arg( i ). Como i = e i ( 2 +2πk) el argumento de i
es el conjunto  π 
arg( i ) = + 2πk | k ∈ Z .
θk :=
2
π
Por definición Arg(z) es lo los únicos valore tales que −π 6 θk < +π, a saber θ0 = 2. Entonces,
Log i = i π2 .
√ √
2+ i Arg(−1+ i ). Como −1+ i = 2e i (π 4 +2πk)
3
(b) Log (−1+ i ) = log |−1+ i |+ i Arg(−1+ i ) = log
el argumento es el conjunto
( )
3
arg(−1 + i ) = θk := π + 2πk | k ∈ Z
4

y Arg(−1 + i ) = π 43 . Entonces, Log (−1 + i ) = log 2 + i π 34 .

(c) Log [ i (−1 + i )] = log | i (−1 + i )| + i Arg[ i (−1 + i )] = log 2 + i Arg(−1 − i ). Como −1 − i =
√ − i (π 3 −2πk)
2e 4 el argumento es el conjunto
( )
3
arg(−1 − i ) = θk := −π + 2πk | k ∈ Z
4

y Arg(−1 − i ) = −π 34 . Entonces, Log [ i (−1 + i )] = log 2 − i π 34 .

(d) Se sigue de (a) y (b) que


π √ 3 √ 5
Log i + Log (−1 + i ) = i + log 2 + i π = log 2 + i π .
2 4 4
Pero de (c) se deduce que

Log i + Log (−1 + i ) , Log [ i (−1 + i )] .

Observar que (Log i + Log (−1 + i )) − Log [ i (−1 + i )] = i 2π. ¿Por qué? 

Ejercicio 2
Sea a ∈ C − {0}. La función potencia compleja za : R → R está definida por

za := ea log z
.
1
(a) Describa la superficie de Riemann modificada R [m] de la función raı́z m-ésima w(z) := z m ,
m ∈ N.
Muestre que:
h p i
(b) arcsin(z) = − i log i z + (1 − z2 ) como mapeo de R [2] en C − {0};
h p i
(c) arccos(z) = − i log z + (z2 − 1) como mapeo de R [2] en C − {0}.
3

Solución.
(a) Sea z = r e i (φ+2πk) , r > 0 y φ ∈ [−π, +π) un punto en R. La raı́z m-ésima de z está dada por
 1 φ  φ 
w(z) = z m := e m log z = e m log |z|+ i (φ+2πk) = elog r m e+ i m +2π m = r m e+ i m +2π m .
1 1 1 k 1 k

φ 
La diferencia en el argumento entre los puntos zk := r m e+ i m +2π m
1 k
, k = 0, 1, . . . , m − 1 es menor
que 2π. Entonces, los m puntos {zk , zk+1 , . . . , zk+m−1 } se pueden considerar todos como puntos de una
1
misma rama de R. Por consiguiente el mapeo w(z) = z m puede visualizarse como un mapeo de R [m]
en C − {0}, donde R [m] consiste en m copias de C − {0} “pegadas” una déspues de otra a lo largo del eje
real negativo, como R, excepto que el borde superior de la rama de ariba se “pega” al borde inferior
de la rama de abajo.


Figure 0.2. La la superficie de Riemann modificada R [2] para w(z) = z.

(b) La función w(z) := arcsin(z) revierte la acción del mapeo


e+ i w − e− i w
z = sin(w) = .
2i
Si multiplicamos ambos extremos de esta ecuación por 2 i e+ i w , tenemos
2
e+ i w − 2 i z e+ i w − 1 = 0 .


La solución está dada por



+iw 2iz ± −4z2 + 4 p
e = = i z ± 1 − z2 .
2
√ √
Se observa que + 1 − z2 = |1 − z2 | e i 2 arg(1−z ) y − 1 − z2 = |1 − z2 | e i 2 arg(1−z )+ i π . Entonces,
p 1 2
p 1 2

obtenemos que
e+ i w = i z +
p
1 − z2 .
4

como mapeo de R [2] en C − {0}. Al tomar logaritmos de ambos lados resulta que
1 h p i
w(z) = log i z + (1 − z2 ) .
i

(c) Como en (b)


e+ i w + e− i w
z = cos(w) = .
2
Si multiplicamos ambos extremos de esta ecuación por 2e+ i w , tenemos
2
e+ i w − 2z e+ i w + 1 = 0 .


La solución está dada por



+iw 2z ± 4z2 − 4 p
e = = z ± z2 − 1 .
2
como mapeo de R [2] en C − {0} obtenemos que

e+ i w = z +
p
z2 − 1 .

y al tomar logaritmos de ambos lados resulta que


1 h p i
w(z) = log z + (z2 − 1) .
i


Ejercicio 3
Sea m ∈ Z, R > 0 y z0 ∈ C.
(a) Prueba que


0

 m , −1
(z − z0 )m dz = 

 i 2π m = −1

|z−z0 |=R 

donde el cı́rculo |z − z0 | = R es parametrizado en el sentido positivo (antihorario).


Consideremos ahora el caso m = −1 y z0 = 0. Recordamos que la función f (z) = z−1 tiene antiderivada
F(z) = log(z) + C.
(b) Muestre que el resultado anterior no está en contradicción con el teorema fundamental del
cálculo.

Solución.
(a) Fijamos la siguiente parametrización de |z − z0 | = R

[0, 2π] 3 t 7→ z(t) := z0 + R e i t .

Resulta que
dz
(t) = i R e i t , t ∈ [0, 2π] .
dt
5

La integral de lı́nea esta dada por


Z 2π
dz
(z − z0 ) m
dz : = (z(t) − z0 )m (t) dt
|z−z0 |=R 0 dt
Z 2π  m
= R eit i R e i t dt
0
Z 2π
= i Rm+1 e i (m+1)t dt .
0

Entonces,
#2π
e i (m+1)t
"
(z − z0 ) m
dz = i R m+1
= 0 si m , −1
|z−z0 |=R i (m + 1) 0
y

(z − z0 )m dz = i R0 [t]2π
0 = i 2π si m = −1 .
|z−z0 |=R

(b) La función logaritmo complejo está definida en la superficie de Riemann R y aquı́ el arco |z| = R
no es cerrado. Una parametrización posible sobre la rama k lo arco es

[−1, +1] 3 t 7→ z(t) := R e i π (t+2k) .

Como log z := log |z| + i arg(z) y arg(z(±1)) = π (±1 + 2k) se deduce que

F(z(+1)) − F(z(−1)) = log |z(+1)| + i arg(z(+1)) − log |z(−1)| − i arg(z(−1))


= log R + i π(+1 + 2k) − log R − i π(−1 + 2k)
= i 2π .

Entonces,

dz
= F(z(+1)) − F(z(−1))
|z|=R z
sobre cada rama. 

Ejercicio 4
Muestre que

ez 2
π e2 .

dz 6
|z−1|=1 z + 3 3

Solución.
Fijamos la siguiente parametrización de |z − 1| = 1

[0, 2π] 3 t 7→ z(t) := 1 + e i t .

Resulta que
dz
(t) = i e i t , t ∈ [0, 2π] .
dt
6

La integral de lı́nea esta dada por



ez ez(t) dz
Z
dz : = (t) dt
|z−1|=1 z+3 0 z(t) + 3 dt
2π it
e1+e
Z
= i e i t dt
0 4 + eit

eit
Z
it
= ie ee dt .
0 4 + eit

Entonces,
Z 2π
ez e i t e i t
dz 6 e e dt
|z−1|=1 z + 3 4 + eit
0
Z 2π
e i t 1
= e e dt
0 |4 + e i t|
Z 2π
e e i t
6 e dt
3 0

dado que |4 + e i t | > 3. Para terminar observamos que


i t cos(t)+ i sin(t) cos(t) i sin(t) cos(t)
ee = e = e e = e 6 e.

Entonces,
Z 2π
ez e2 2
dt = π e2 .

dz 6
|z−1|=1 z + 3 3 0 3



Ejercicio 5
Muestre que en la rama principal (k = 0)

log z √ log R
2
dz 6 2π 2
|z|=R z R

si R > eπ .

Solución.
La función logaritmo complejo está definida en la superficie de Riemann R. Sobre la rama k el arco
(no cerrado) |z| = R de se parametriza como

[−1, +1] 3 t 7→ z(t) := R e i π (t+2k) .

Observamos que arg(z(−1)) = 2πk − π y arg(z(+1)) = 2πk + π. Además

dz
(t) = i π R e i π (t+2k) , t ∈ [−1, +1] .
dt
7

La integral de lı́nea esta dada por


Z +1
log z log |z(t)| + i arg(z(t)) dz
2
dz : = (t) dt
|z|=R z −1 z(t)2 dt
Z +1
log R + i π(t + 2k)
= 2 e i 2π (t+2k)
i π R e i π (t+2k) dt
−1 R
Z +1
π log R + i π(t + 2k)
= i dt .
R −1 e i π (t+2k)
Entonces, Z +1
π log R + i π(t + 2k) dt

log z
dz 6
|z|=R z
2 R −1 e i π (t+2k)

Z +1
π log R + i π(t + 2k) dt
=
R −1
Z +1 q
π
= (log R)2 + π2 (t + 2k)2 dt
R −1
s
Z +1 !2
log R π(t + 2k)
= π 1 + dt .
R −1 log R
Si R > eπ resulta que
π(t + 2k)
6 (t + 2k) 6 1 + 2k , ∀ t ∈ [−1, +1].
log R
Entonces, q
log z log R
1 + (1 + 2k)2

2
dz 6 2π
|z|=R z R

Para terminar recordamos que la la rama principal es definita para k = 0. 

También podría gustarte